Znaleziono 816 wyników

autor: PoweredDragon
11 mar 2020, o 17:30
Forum: Funkcje trygonometryczne i cyklometryczne
Temat: Naszkicuj wykres funkcji cos x do potęgi
Odpowiedzi: 11
Odsłony: 2957

Re: Naszkicuj wykres funkcji cos x do potęgi

Dla x=0 dostajemy wyrażenie 0^0 , któe nie ma sensu. Z tego powodu zero należy wykluczyć z dziedziny. Dlaczego dla x=0 otrzymujemy wyrażenie 0^0 , a nie 1^0 , skoro cos 0 = 1 ? Prawdopodobnie dlatego, że tam jest \cos( x^{\text{cośtam}} ) , a nie (\cos x)^{\text{cośtam}} ; dokładnie ten sam błąd po...
autor: PoweredDragon
25 mar 2019, o 20:23
Forum: Rachunek całkowy
Temat: Całka z pierwiastkiem 4 stopnia
Odpowiedzi: 1
Odsłony: 519

Re: Całka z pierwiastkiem 4 stopnia

Podstawienie jak najbardziej dobre
autor: PoweredDragon
25 mar 2019, o 19:11
Forum: Rachunek różniczkowy
Temat: Udowodnij nierówność wykorzystując rachunek różniczkowy
Odpowiedzi: 3
Odsłony: 764

Re: Udowodnij nierówność wykorzystując rachunek różniczkowy

Kilka schematów można znaleźć f(x) \ge g(x) \\ h(x) := f(x)-g(x) \ge 0 I można np. pokazać, że w punkcie x_0 jest równość; na lewo od x_0 funkcja h jest malejąca, a na prawo rosnąca. (w szczególności x_0 =0 i h może być parzysta tak jak tutaj ) Można np. pokazać, że h ma ekstrema lokalne i każde z n...
autor: PoweredDragon
23 mar 2019, o 19:30
Forum: Pytania, uwagi, komentarze...
Temat: Gdzie są TK?
Odpowiedzi: 1
Odsłony: 2191

Gdzie są TK?

W jakim dziale możnaby umieszczać pytania o Teorię Klas (choć ta mniej) czy Teorię Kategorii? Wydaje mi się, że są dość popularne wśród studentów (szczególnie ta druga) i są jednymi z tych (znowu, ta druga głównie), z którymi studenci mają najwięcej problemów lub co do których mają najwięcej wątpliw...
autor: PoweredDragon
13 lut 2019, o 14:32
Forum: Kółko matematyczne
Temat: [MIX] Mix matematyczny 38
Odpowiedzi: 15
Odsłony: 2899

Re: [MIX] Mix matematyczny 38

Interesują nas rozwiązania x \ge - \frac{1}{3} (x+1)^4=(3x+1)^3 x^4-23x^3-21x^2-5x=0 x(x^3-23x^2-21x-5)=0 Niech W(x) = x^3-23x^2-21x-5 , wówczas równanie x^3-23x^2-21x-5 = 0 ma dokładnie jeden pierwiastek dodatni (na mocy reguły Kartezjusza) Dla x = -\frac{1}{3} mamy W(x) = -\frac{176}{27} Zaś z an...
autor: PoweredDragon
10 sty 2019, o 23:54
Forum: Teoria liczb
Temat: Potęga pi
Odpowiedzi: 6
Odsłony: 1635

Re: Potęga pi

Nie wiem czemu ucięty jest fragment. Miało być "czyli w szczególności dla singletonów liczb algebraicznych \(\displaystyle{ \left\{ \alpha\right\}}\), \(\displaystyle{ e^{\alpha}}\) jest przestępne"
autor: PoweredDragon
10 sty 2019, o 17:50
Forum: Funkcje trygonometryczne i cyklometryczne
Temat: Tożsamość trygonometryczna
Odpowiedzi: 1
Odsłony: 645

Tożsamość trygonometryczna

\(\displaystyle{ \tg \frac{\alpha}{2} = \frac{\sin \frac{\alpha}{2}}{\cos \frac{\alpha}{2}}}\)
Z wzorów na \(\displaystyle{ \cos 2 \alpha}\) i \(\displaystyle{ \sin 2 \alpha}\)
\(\displaystyle{ \sin \alpha = 2 \sin \frac{\alpha}{2} \cos \frac{\alpha}{2}}\)
\(\displaystyle{ \cos \alpha = 2 \cos^2 \frac{\alpha}{2} - 1}\)
autor: PoweredDragon
6 sty 2019, o 16:51
Forum: Algebra abstrakcyjna
Temat: Czy cykle tworzą grupę?
Odpowiedzi: 7
Odsłony: 1160

Re: Czy cykle tworzą grupę?

Pewnie znając mnie nie, ale ja kuleję z dowodami algebraicznymi dużo bardziej niż z intuicją (która równie często zawodzi). Pewnie to było coś na zasadzie przedstawiania iloczynów a^{k_1} b^{l_1} a^{k_2} b^{l_3} ... a^{k_n} b^{l_n} I jakoś tam domnażało się prawo i lewo stronnie, żeby otrzymywać jed...
autor: PoweredDragon
6 sty 2019, o 15:55
Forum: Algebra abstrakcyjna
Temat: Czy cykle tworzą grupę?
Odpowiedzi: 7
Odsłony: 1160

Re: Czy cykle tworzą grupę?

Grupa generowana przez elementy a i b , to grupa elementów postaci a^n b^m gdzie potęgowanie jest ze względu na działanie wewnętrzne w grupie. m, n \in \mathbb Z . Tak jest tylko dla grup przemiennych. Faktycznie. Zapomniałem kompletnie o przemienności No to zostaje chyba kwestia domnażania z prawe...
autor: PoweredDragon
5 sty 2019, o 23:40
Forum: Geometria trójkąta
Temat: Twierdzenie odwrotne do tego o kącie środkowym i wpisanym
Odpowiedzi: 2
Odsłony: 1232

Re: Twierdzenie odwrotne do tego o kącie środkowym i wpisany

Takie tw. nie zachodzi. Wstęp do dowodu:

Narysuj okrąg. Oznacz łuk \(\displaystyle{ AB}\). Opisz okrąg na trójkącie \(\displaystyle{ ABO}\) (\(\displaystyle{ O}\) - środek tego pierwszego okręgu). Wniosek powinien nasunąć się sam.
autor: PoweredDragon
5 sty 2019, o 16:30
Forum: Funkcje logarytmiczne i wykładnicze
Temat: Funkcja wykładniczna- zd. z parametrem
Odpowiedzi: 3
Odsłony: 869

Re: Funkcja wykładniczna- zd. z parametrem

To to zrób i sprawdź kiedy pozioma prosta przecina go w dokładnie dwóch miejscach
autor: PoweredDragon
5 sty 2019, o 15:14
Forum: Statystyka
Temat: Przedziały ufności
Odpowiedzi: 5
Odsłony: 2392

Przedziały ufności

qurczaq pisze:Staram się zrozumieć treść wykładów, ale matematyka nie jest moją pasją i mam z tym trudność, więc pytam lepszych od siebie o pomoc.
A co studiujesz?
autor: PoweredDragon
5 sty 2019, o 15:07
Forum: Algebra abstrakcyjna
Temat: Czy cykle tworzą grupę?
Odpowiedzi: 7
Odsłony: 1160

Re: Czy cykle tworzą grupę?

No z definicji generatorów. Grupa generowana przez elementy a i b , to grupa elementów postaci a^n b^m gdzie potęgowanie jest ze względu na działanie wewnętrzne w grupie. m, n \in \mathbb Z . Pytanie można zadać, czy złożenia \gamma^n \sigma^m generują wszystkie elementy S_6 i tylko elementy S_6
autor: PoweredDragon
5 sty 2019, o 14:06
Forum: Statystyka
Temat: mocne prawo wielkich liczb
Odpowiedzi: 3
Odsłony: 826

Re: mocne prawo wielkich liczb

Jakieś próby rozwiązania?
autor: PoweredDragon
5 sty 2019, o 13:57
Forum: Inne funkcje + ogólne własności
Temat: Udowodnić, że symetryczna
Odpowiedzi: 3
Odsłony: 863

Re: Udowodnić, że symetryczna

A symetryczna to nie oznacza, że dla funkcji wieloargumentowej, wartość jest taka sama niezależnie od permutacji tego samego zbioru argumentów? Wówczas dla \alpha=\beta mamy f(\alpha, \beta) = f(\alpha, \alpha) = f(\beta, \alpha) dla dowolnej funkcji dwuargumentowej... To raczej nie ta definicja, al...